Exam 4 Flashcards

1
Q

Which of the following best explains the mechanism by which estradiol (E2) induces the luteinizing hormone (LH) surge?

A) Estradiol continuously inhibits GnRH, which suddenly stops at ovulation, causing an LH spike.
B) Estradiol shifts from negative to positive feedback on the hypothalamus and anterior pituitary, leading to increased GnRH and LH secretion.
C) Progesterone levels rise before ovulation, triggering a temporary surge in LH secretion.
D) The anterior pituitary secretes LH independently of hormonal regulation when estrogen levels are high.
E) LH levels gradually increase throughout the menstrual cycle without a distinct surge.

A

Answer: B
✅ Rationale: Estradiol (E2) normally inhibits GnRH and gonadotropin release via negative feedback. However, around day 14 of the menstrual cycle, persistent high estradiol levels switch to positive feedback, leading to a sharp LH surge that triggers ovulation.

How well did you know this?
1
Not at all
2
3
4
5
Perfectly
2
Q

At what point in the menstrual cycle does the LH surge typically occur, and what event follows this surge?

A) Day 7–10; Follicular phase initiation
B) Day 12–16; Ovulation within 24–36 hours
C) Day 18–22; Corpus luteum formation
D) Day 1–5; Shedding of the endometrial lining
E) Day 25–28; Onset of menstruation

A

Answer: B
✅ Rationale: The LH surge occurs around days 12–16 of the menstrual cycle, peaking approximately 36 hours before ovulation. This surge triggers follicular rupture, releasing the oocyte for fertilization.

How well did you know this?
1
Not at all
2
3
4
5
Perfectly
3
Q

Which of the following conditions is most likely to disrupt the LH surge, leading to anovulation?

A) Polycystic ovary syndrome (PCOS)
B) Hyperprolactinemia
C) Functional hypothalamic amenorrhea
D) All of the above
E) None of the above

A

Answer: D
✅ Rationale: PCOS (A) disrupts the LH surge by causing chronic LH elevation without a proper surge. Hyperprolactinemia (B) suppresses GnRH secretion, impairing LH secretion. Functional hypothalamic amenorrhea (C) results from low energy availability or stress, leading to low GnRH, FSH, and LH secretion, preventing ovulation.

How well did you know this?
1
Not at all
2
3
4
5
Perfectly
4
Q

Question 1: Regulation of GnRH Release
Which of the following conditions would most likely lead to increased GnRH release?

A) High estradiol and progesterone levels during the luteal phase
B) Low estrogen and progesterone levels following menstruation
C) Chronic administration of exogenous estrogen
D) Persistent hyperprolactinemia
E) Administration of a GnRH antagonist

A

Answer: B
✅ Rationale: GnRH is released from the hypothalamus in response to low estrogen and progesterone, which occurs after menstruation. High levels of sex steroids (A, C) inhibit GnRH via negative feedback. Hyperprolactinemia (D) suppresses GnRH, and GnRH antagonists (E) directly block its action.

How well did you know this?
1
Not at all
2
3
4
5
Perfectly
5
Q

Question 2: Mechanism of Gonadotropin Release
Which of the following statements best describes the role of gonadotropins?

A) LH stimulates follicular development, while FSH triggers ovulation.
B) FSH and LH are secreted by the hypothalamus and act on the pituitary gland.
C) GnRH stimulates the anterior pituitary to release FSH and LH, which act on the ovaries.
D) Inhibins selectively increase LH secretion but decrease FSH levels.
E) FSH is the primary driver of ovulation, while LH maintains corpus luteum function.

A

Answer: C
✅ Rationale: GnRH stimulates the anterior pituitary to release FSH and LH, which stimulate ovarian follicle development and ovulation, respectively. (A) is incorrect because FSH promotes follicular growth, while LH triggers ovulation. (B) is incorrect because FSH & LH come from the anterior pituitary, not the hypothalamus. (D) is incorrect because inhibins selectively inhibit FSH, not increase LH. (E) is incorrect because LH triggers ovulation, not FSH.

How well did you know this?
1
Not at all
2
3
4
5
Perfectly
6
Q

Timing of LH and Ovulation
During the menstrual cycle, when does the LH surge most likely occur, and what is its primary function?

A) Days 5–7; Stimulates follicular development
B) Days 12–16; Triggers ovulation
C) Days 18–22; Prepares the uterus for implantation
D) Days 25–28; Induces endometrial shedding
E) Days 1–3; Suppresses FSH release

A

Answer: B
✅ Rationale: The LH surge occurs around days 12–16, causing follicular rupture and ovulation. (A) is incorrect because FSH drives early follicular development. (C) is incorrect because post-ovulation (luteal phase) supports implantation, but LH surge is before that. (D) is incorrect because menstruation occurs due to hormonal withdrawal, not LH. (E) is incorrect because LH does not suppress FSH—Inhibins do.

How well did you know this?
1
Not at all
2
3
4
5
Perfectly
7
Q

Role of FSH in Follicular Development
Which of the following best describes the function of FSH?

A) Directly triggers ovulation
B) Stimulates ovarian follicles to grow and produce estradiol
C) Inhibits LH release to maintain follicular dominance
D) Suppresses progesterone secretion in the luteal phase
E) Selectively inhibits the production of inhibins

A

Answer: B
✅ Rationale: FSH stimulates follicular growth and estradiol production. (A) is incorrect because LH, not FSH, triggers ovulation. (C) is incorrect because FSH and LH work together, and inhibins suppress FSH. (D) is incorrect because progesterone is regulated by the corpus luteum, not FSH. (E) is incorrect because inhibins inhibit FSH, not the other way around.

How well did you know this?
1
Not at all
2
3
4
5
Perfectly
8
Q

What is the primary role of progesterone during the secretory phase of the menstrual cycle?

A) Stimulates follicular development and estradiol production
B) Induces the LH surge necessary for ovulation
C) Maintains the endometrial lining by promoting glandular secretions and vascularization
D) Directly suppresses FSH to prevent ovulation
E) Increases GnRH secretion to prolong the luteal phase

A

Answer: C
✅ Rationale: During the secretory phase (days 14–28, post-ovulation), progesterone is secreted by the corpus luteum to stabilize the endometrial lining, making it receptive to implantation by increasing glandular secretions and blood vessel formation. (A) is incorrect because FSH, not progesterone, stimulates follicular growth. (B) is incorrect because LH surge occurs before the secretory phase. (D) is partially true, but progesterone’s main role is endometrial maintenance. (E) is incorrect because progesterone inhibits, not increases, GnRH.

How well did you know this?
1
Not at all
2
3
4
5
Perfectly
9
Q

What is the most direct consequence of progesterone withdrawal at the end of the secretory phase if implantation does not occur?

A) FSH levels surge, leading to immediate follicular recruitment
B) Spiral artery constriction leads to endometrial shedding (menstruation)
C) Increased estrogen levels prolong the luteal phase
D) Inhibins prevent FSH and LH release, maintaining the corpus luteum
E) GnRH secretion is inhibited, leading to an extended cycle

A

Answer: B
✅ Rationale: If fertilization does not occur, the corpus luteum degenerates, leading to a rapid drop in progesterone. This causes spiral artery constriction, leading to endometrial ischemia, necrosis, and shedding—triggering menstruation. (A) is incorrect because FSH levels remain low until menstruation completes. (C) is incorrect because estrogen declines with progesterone. (D) is incorrect because without pregnancy, the corpus luteum degenerates. (E) is incorrect because GnRH resumes secretion to initiate a new cycle.

How well did you know this?
1
Not at all
2
3
4
5
Perfectly
10
Q

Which of the following best explains why progesterone causes a small rise in body temperature after ovulation?

A) Progesterone directly stimulates the anterior pituitary to increase metabolic rate.
B) Progesterone interacts with the hypothalamic thermoregulatory center, increasing basal body temperature.
C) Progesterone induces thyroid hormone release, which increases core body temperature.
D) Increased progesterone reduces LH secretion, causing a temporary rise in heat production.
E) Progesterone stimulates uterine contractions, generating internal heat.

A

Answer: B
✅ Rationale: Progesterone increases basal body temperature by acting on the hypothalamic thermoregulatory center. This post-ovulatory temperature rise is often used in fertility tracking. (A) is incorrect because progesterone does not directly affect metabolic rate via the anterior pituitary. (C) is incorrect because progesterone does not induce thyroid hormone release. (D) is incorrect because LH suppression does not regulate body temperature. (E) is incorrect because uterine contractions do not significantly contribute to overall body heat.

How well did you know this?
1
Not at all
2
3
4
5
Perfectly
11
Q

When is the most optimal time for sexual intercourse to maximize the chances of fertilization?

A) 5–7 days before ovulation
B) 3–5 days after ovulation
C) 1–2 days before ovulation and on the day of ovulation
D) The day after ovulation only
E) Anytime during the luteal phase

A

Answer: C
✅ Rationale: Sperm can survive up to 5 days, but their optimal viability is 1–2 days. The highest probability of conception occurs when sperm are present in the reproductive tract just before ovulation. (A) is incorrect because sperm viability declines after 5 days. (B) and (D) are incorrect because the egg is viable for only 12–24 hours after ovulation. (E) is incorrect because the luteal phase is post-ovulation, meaning fertilization is unlikely.

How well did you know this?
1
Not at all
2
3
4
5
Perfectly
12
Q

Which of the following statements best describes the role of estriol (E3) in pregnancy?

A) Estriol is primarily produced by the corpus luteum throughout pregnancy.
B) Estriol (E3) is the dominant estrogen during the 2nd and 3rd trimesters, synthesized by the placenta.
C) Estriol is produced in large amounts in early pregnancy to stimulate the corpus luteum.
D) Estriol (E3) is secreted by the anterior pituitary in response to rising progesterone levels.
E) Estriol has no significant role in pregnancy and is replaced entirely by estradiol (E2).

A

Answer: B
✅ Rationale: Estriol (E3) is the primary estrogen during the 2nd and 3rd trimesters of pregnancy and is produced by the placenta. (A) is incorrect because the corpus luteum primarily produces progesterone and estradiol (E2) early in pregnancy, not E3. (C) is incorrect because E3 levels rise significantly only after the placenta takes over hormone production. (D) is incorrect because the anterior pituitary does not secrete E3. (E) is incorrect because estriol is the dominant estrogen during later pregnancy.

How well did you know this?
1
Not at all
2
3
4
5
Perfectly
13
Q

What is the primary function of human chorionic gonadotropin (hCG) in early pregnancy?

A) Stimulates the corpus luteum to continue producing progesterone until the placenta takes over
B) Directly stimulates estriol (E3) production by the placenta
C) Initiates the second trimester by triggering the switch from corpus luteum to placental hormone production
D) Suppresses FSH and LH secretion throughout pregnancy
E) Acts as the primary estrogen regulator during the third trimester

A

Answer: A
✅ Rationale: hCG is secreted by the placenta and functions to “rescue” the corpus luteum, ensuring continued progesterone production to maintain early pregnancy until the placenta fully takes over. (B) is incorrect because E3 is produced later by the placenta, not regulated by hCG. (C) is incorrect because hCG maintains the corpus luteum but does not directly trigger the placental transition. (D) is partially true (hCG has minor LH suppression effects), but its primary function is corpus luteum maintenance. (E) is incorrect because estrogen regulation shifts to the placenta in later pregnancy, and hCG does not control this process.

How well did you know this?
1
Not at all
2
3
4
5
Perfectly
14
Q

Which of the following estrogens is the most potent at estrogen receptors?

A) Estrone (E1)
B) Estradiol (E2)
C) Estriol (E3)
D) Equilin
E) Ethinyl estradiol

A

Answer: B
✅ Rationale: Estradiol (E2) is the most potent natural estrogen at the receptor level. Estrone (A) is weaker but can be converted to estradiol locally. Estriol (C) is the weakest and primarily produced in pregnancy. Equilin (D) is a component of conjugated estrogens. Ethinyl estradiol (E) is a synthetic estrogen with high bioavailability but lower receptor potency than E2.

How well did you know this?
1
Not at all
2
3
4
5
Perfectly
15
Q

Which estrogen best survives oral delivery, making it more bioavailable when taken orally?

A) Estradiol (E2)
B) Estrone (E1)
C) Estriol (E3)
D) Ethinyl estradiol
E) Progesterone

A

Answer: D
✅ Rationale: Ethinyl estradiol is the most bioavailable when taken orally due to its resistance to first-pass metabolism. Estriol (C) also survives oral delivery better than estradiol but is weaker at the receptor. Estradiol (A) undergoes significant first-pass metabolism, reducing its oral bioavailability. Estrone (B) is weak and primarily interconverted with E2. Progesterone (E) is not an estrogen.

How well did you know this?
1
Not at all
2
3
4
5
Perfectly
16
Q

Look at these three estrogen sisters: Estrone, Estradiol, and Estriol.

One of them is the strongest and has two big OH (hydroxyl) groups—one at the top and one at the bottom!

Which one is the strongest big OH sister?

A) Estrone (E1) – Only one OH, kinda weak
B) Estradiol (E2) – Two OHs, super strong!
C) Estriol (E3) – Three OHs, but super weak
D) Testosterone – Wrong family, dude
E) Cholesterol – What are you even doing here?!

A

Answer: B – Estradiol!
✅ Rationale: Estradiol (E2) is the strongest at estrogen receptors and has two hydroxyl (-OH) groups—one on the A-ring and one on the D-ring. Estrone (E1) only has one OH, and Estriol (E3) has three but is weaker. So if you see two OH groups, that’s Big Boss Estradiol!

How well did you know this?
1
Not at all
2
3
4
5
Perfectly
17
Q

What triggers the release of gonadotropin-releasing hormone (GnRH) from the hypothalamus?

A) High estrogen (E2) and progesterone (P) levels
B) Low estrogen and progesterone levels
C) Direct stimulation by LH and FSH
D) Androgen binding to the anterior pituitary
E) Continuous secretion independent of hormonal regulation

A

Answer: B
✅ Rationale: GnRH is released from the hypothalamus when estrogen (E2) and progesterone (P) levels are low. This stimulates the anterior pituitary to release FSH and LH, promoting follicular growth. (A) is incorrect because high estrogen/progesterone suppress GnRH release. (C) is incorrect because FSH & LH are stimulated by GnRH, not the other way around. (D) is incorrect because androgens do not directly regulate GnRH. (E) is incorrect because GnRH secretion is pulsatile and regulated by feedback loops.

How well did you know this?
1
Not at all
2
3
4
5
Perfectly
18
Q

Which of the following describes the primary function of luteinizing hormone (LH) in ovarian steroid production?

A) Stimulates granulosa cells to convert androgens into estradiol via aromatase
B) Acts on theca cells to synthesize testosterone and androstenedione
C) Directly induces progesterone secretion from granulosa cells before ovulation
D) Inhibits FSH secretion to regulate estrogen production
E) Stimulates corpus luteum formation but plays no role in steroid synthesis

A

Answer: B
✅ Rationale: LH stimulates theca cells to produce testosterone (T) and androstenedione (A4), which serve as precursors for estrogen synthesis in granulosa cells. (A) is incorrect because granulosa cells use aromatase to convert androgens into estrogens, but this is stimulated by FSH, not LH. (C) is incorrect because progesterone is secreted after ovulation, not before. (D) is incorrect because LH does not directly inhibit FSH secretion. (E) is incorrect because LH is critical for steroid production.

How well did you know this?
1
Not at all
2
3
4
5
Perfectly
19
Q

Which of the following correctly describes follicle-stimulating hormone (FSH) function in ovarian hormone synthesis?

A) Stimulates granulosa cells to convert testosterone & androstenedione into estradiol and estrone
B) Increases progesterone secretion in theca cells before ovulation
C) Directly promotes ovulation alongside LH
D) Inhibits androgen production in theca cells
E) Converts estrogen into androgens via aromatase enzymes

A

Answer: A
✅ Rationale: FSH acts on granulosa cells, stimulating aromatase activity to convert testosterone (T) and androstenedione (A4) into estradiol (E2) and estrone (E1). (B) is incorrect because theca cells primarily produce androgens, not progesterone. (C) is incorrect because LH, not FSH, is the key hormone triggering ovulation. (D) is incorrect because FSH does not inhibit theca cell androgen production—LH stimulates it. (E) is incorrect because aromatase converts androgens into estrogens, not the other way around.

How well did you know this?
1
Not at all
2
3
4
5
Perfectly
20
Q

Which of the following best describes the primary function of estradiol (E2) and progesterone (P) during the menstrual cycle?

A) Estradiol (E2) prepares the endometrium for implantation, while progesterone (P) maintains it.
B) Estradiol (E2) inhibits FSH & LH release, while progesterone (P) stimulates ovulation.
C) Estradiol (E2) promotes corpus luteum formation, while progesterone (P) increases GnRH secretion.
D) Estradiol (E2) stimulates theca cells, while progesterone (P) stimulates granulosa cells.
E) Estradiol (E2) decreases after ovulation, while progesterone (P) remains low throughout the cycle.

A

Answer: A
✅ Rationale: Estradiol (E2) thickens the endometrial lining to prepare for implantation, while progesterone (P) maintains it post-ovulation. (B) is incorrect because estradiol has both inhibitory and stimulatory effects on LH & FSH depending on the phase. (C) is incorrect because LH, not estradiol, promotes corpus luteum formation. (D) is incorrect because FSH acts on granulosa cells, and LH acts on theca cells. (E) is incorrect because progesterone surges post-ovulation in the luteal phase.

How well did you know this?
1
Not at all
2
3
4
5
Perfectly
21
Q

Which of the following best describes the negative feedback regulation of the hypothalamic-pituitary-ovarian (HPO) axis?

A) Estradiol (E2) and progesterone inhibit GnRH, LH, and FSH release to regulate hormone balance.
B) LH surge causes a continuous increase in GnRH secretion, stimulating ovulation.
C) The anterior pituitary secretes GnRH, which stimulates estrogen production.
D) Progesterone stimulates FSH secretion to maintain the corpus luteum.
E) Theca cells provide direct negative feedback to granulosa cells, inhibiting steroidogenesis.

A

Answer: A
✅ Rationale: The HPO axis operates under negative feedback regulation, where estradiol (E2) and progesterone inhibit GnRH, LH, and FSH secretion to maintain homeostasis. (B) is incorrect because the LH surge is a positive feedback loop. (C) is incorrect because GnRH is secreted by the hypothalamus, not the anterior pituitary. (D) is incorrect because progesterone inhibits FSH to prevent multiple ovulations. (E) is incorrect because theca cells and granulosa cells work together, not via direct inhibition.

How well did you know this?
1
Not at all
2
3
4
5
Perfectly
22
Q

Which of the following triggers the LH surge, leading to ovulation?

A) Persistent high levels of estradiol (E2) from a mature follicle switch feedback from negative to positive.
B) A sudden drop in progesterone allows the anterior pituitary to release LH.
C) Constant inhibition of FSH leads to a compensatory LH increase.
D) Low estrogen and progesterone levels stimulate GnRH secretion, increasing LH release.
E) The corpus luteum secretes inhibins, which drive the LH surge.

A

Answer: A
✅ Rationale: The LH surge is a positive feedback event, where high estradiol (E2) levels from a mature follicle cause a switch from negative to positive feedback, leading to a sharp rise in LH and ovulation. (B) is incorrect because progesterone drops after ovulation, not before. (C) is incorrect because FSH inhibition does not directly cause an LH surge. (D) is incorrect because low estrogen and progesterone typically inhibit GnRH, not trigger an LH surge. (E) is incorrect because inhibins selectively inhibit FSH, not drive LH release.

How well did you know this?
1
Not at all
2
3
4
5
Perfectly
23
Q

How does GnRH pulse frequency change during the follicular phase, and what effect does this have?

A) GnRH pulses remain steady at 90-minute intervals, maintaining constant FSH and LH secretion.
B) Early follicular phase: GnRH pulses every 90 minutes; late follicular phase: GnRH pulses every 60 minutes, increasing LH secretion and estradiol (E2) production.
C) GnRH pulses become slower throughout the follicular phase, suppressing FSH and LH.
D) LH secretion is highest during the early follicular phase and declines in the late follicular phase.
E) Estradiol secretion suppresses GnRH pulses throughout the follicular phase.

A

Answer: B
✅ Rationale: During the early follicular phase, GnRH pulses at 90-minute intervals, maintaining steady FSH and LH secretion. In the late follicular phase, GnRH pulses accelerate to every 60 minutes, leading to a spike in LH secretion and estradiol production, which prepares for ovulation. (A) is incorrect because pulse frequency changes in the late phase. (C) is incorrect because pulse frequency increases, not slows down. (D) is incorrect because LH secretion rises in the late follicular phase, not early. (E) is incorrect because estradiol increases GnRH pulse frequency rather than suppressing it.

How well did you know this?
1
Not at all
2
3
4
5
Perfectly
24
Q

What is the primary role of FSH during the early follicular phase?

A) Stimulates granulosa cells to convert testosterone into estradiol (E2).
B) Directly triggers ovulation in response to the LH surge.
C) Inhibits dominant follicle selection to prevent early ovulation.
D) Suppresses GnRH secretion to maintain homeostasis.
E) Stimulates the corpus luteum to maintain progesterone secretion.

A

Answer: A
✅ Rationale: FSH stimulates granulosa cells to convert androgens (testosterone and androstenedione) into estradiol (E2) via aromatase. This rising estradiol level promotes follicular growth and increases GnRH pulse frequency. (B) is incorrect because FSH does not trigger ovulation—LH does. (C) is incorrect because FSH helps select the dominant follicle, not inhibit it. (D) is incorrect because GnRH secretion is controlled by negative feedback from estrogen and progesterone, not FSH. (E) is incorrect because FSH does not maintain the corpus luteum; LH and hCG do.

How well did you know this?
1
Not at all
2
3
4
5
Perfectly
25
Which of the following best describes the role of estradiol (E2) in the proliferative phase of the uterine cycle? A) Stimulates endometrial thickening and vascularization in preparation for potential implantation. B) Suppresses FSH to maintain progesterone secretion and prevent ovulation. C) Directly stimulates LH secretion to trigger ovulation. D) Prevents the corpus luteum from degenerating, ensuring continued progesterone production. E) Causes spiral artery constriction, leading to endometrial shedding.
Answer: A ✅ Rationale: Estradiol (E2) is dominant in the follicular phase and promotes the proliferation of the endometrial lining, thickening it in preparation for possible implantation. (B) is incorrect because FSH is suppressed by inhibins, not directly by E2. (C) is incorrect because E2 does not directly stimulate ovulation—LH surge does. (D) is incorrect because the corpus luteum only forms after ovulation. (E) is incorrect because spiral artery constriction occurs due to progesterone withdrawal, not estradiol.
26
Which of the following best describes the function of the corpus luteum during the luteal phase? A) Produces progesterone, which maintains the endometrium and prevents a new cycle from starting. B) Secretes estradiol (E2) as the dominant hormone, stimulating endometrial proliferation. C) Triggers the LH surge to induce ovulation. D) Secretes high levels of FSH to promote follicular development. E) Decreases progesterone levels, leading to continued GnRH secretion.
Answer: A ✅ Rationale: After ovulation, the ruptured follicle becomes the corpus luteum, which secretes progesterone to maintain the endometrial lining and suppress GnRH, preventing a new cycle from starting. (B) is incorrect because estradiol is dominant in the follicular phase, not the luteal phase. (C) is incorrect because LH surge occurs before ovulation, not during the luteal phase. (D) is incorrect because FSH is suppressed in the luteal phase. (E) is incorrect because progesterone inhibits GnRH to prevent premature follicular recruitment.
27
How do GnRH pulse frequencies change in the luteal phase compared to the follicular phase? A) GnRH pulses slow to every 4 hours, leading to sustained LH secretion and progesterone dominance. B) GnRH pulses increase to every 30 minutes, causing rapid LH and FSH release. C) GnRH pulses remain at 60-minute intervals, maintaining estradiol production. D) GnRH is completely shut down, preventing any hormonal fluctuations. E) GnRH pulses increase to every 90 minutes, stimulating corpus luteum formation.
Answer: A ✅ Rationale: In the luteal phase, progesterone exerts strong negative feedback, slowing GnRH pulses to every 4 hours. This maintains LH secretion but prevents a new follicular cycle from starting. (B) is incorrect because pulse frequency decreases, not increases. (C) is incorrect because estradiol is not the dominant hormone in the luteal phase. (D) is incorrect because GnRH is reduced but not completely shut down. (E) is incorrect because 90-minute pulses occur in the early follicular phase, not the luteal phase.
28
What physiological change occurs due to progesterone secretion in the luteal phase? A) A small rise in basal body temperature due to hypothalamic thermoregulation. B) A sharp drop in body temperature to preserve endometrial stability. C) A decrease in LH secretion, causing body temperature to remain constant. D) Increased FSH secretion, leading to temperature fluctuations. E) Direct stimulation of GnRH release, leading to increased metabolic rate.
Answer: A ✅ Rationale: Progesterone causes a small rise in basal body temperature after ovulation by acting on the hypothalamic thermoregulatory center. (B) is incorrect because progesterone slightly increases, not decreases, temperature. (C) is incorrect because LH remains stable due to slow GnRH pulses. (D) is incorrect because FSH is inhibited in the luteal phase. (E) is incorrect because progesterone inhibits, not stimulates, GnRH release.
29
If fertilization does NOT occur, what happens to the corpus luteum, and what is the physiological consequence? A) The corpus luteum continues producing progesterone, maintaining the endometrium for another cycle. B) The corpus luteum degenerates, leading to a drop in progesterone and initiation of menstruation. C) The corpus luteum converts into theca cells, which restart the follicular phase. D) The corpus luteum remains functional but starts producing FSH instead of progesterone. E) The corpus luteum remains indefinitely, preventing future ovulations.
Answer: B ✅ Rationale: If fertilization does not occur, the corpus luteum degenerates, causing a drop in progesterone levels. This leads to endometrial shedding and menstruation. (A) is incorrect because without hCG stimulation, the corpus luteum does not persist. (C) is incorrect because theca cells do not form from the corpus luteum. (D) is incorrect because FSH is secreted by the anterior pituitary, not the corpus luteum. (E) is incorrect because the corpus luteum has a short lifespan unless pregnancy occurs.
30
If fertilization does occur, which of the following ensures corpus luteum survival and maintenance of pregnancy? A) A surge in LH secretion prolongs corpus luteum function. B) FSH stimulates granulosa cells to continue producing estradiol (E2). C) hCG from the implanted embryo signals the corpus luteum to continue secreting progesterone (P) and estradiol (E2). D) Inhibin prevents corpus luteum degeneration by suppressing FSH and LH. E) Rising progesterone levels stimulate new follicular growth, leading to continued estrogen secretion.
Answer: C ✅ Rationale: hCG (human chorionic gonadotropin) is secreted by the embryo after implantation and "rescues" the corpus luteum, allowing it to continue secreting progesterone (P) and estradiol (E2) to maintain the pregnancy. (A) is incorrect because LH levels fall after ovulation. (B) is incorrect because granulosa cells primarily act during follicular development, not pregnancy maintenance. (D) is incorrect because inhibin mainly suppresses FSH, not corpus luteum degeneration. (E) is incorrect because progesterone inhibits new follicular growth to prevent additional ovulations during pregnancy.
31
How do hormonal contraceptives prevent ovulation? A) They mimic the luteal phase and pregnancy, maintaining negative feedback to suppress GnRH, LH, and FSH secretion. B) They provide continuous GnRH stimulation, leading to increased FSH secretion and follicle development. C) They promote positive feedback on the hypothalamus, increasing LH secretion and triggering ovulation. D) They enhance pulsatile GnRH release, causing continuous follicular maturation. E) They directly inhibit early folliculogenesis, preventing any follicle formation.
Answer: A ✅ Rationale: Hormonal contraceptives mimic the negative feedback seen in the luteal phase and pregnancy, suppressing GnRH, FSH, and LH secretion. This prevents ovulation by inhibiting the LH surge. (B) is incorrect because continuous GnRH stimulation leads to receptor downregulation, not increased FSH. (C) is incorrect because contraceptives inhibit, rather than enhance, GnRH secretion. (D) is incorrect because GnRH pulsatility is reduced, not enhanced. (E) is incorrect because early folliculogenesis is gonadotropin-independent and still occurs; only final follicular selection is inhibited.
32
Why do hormonal contraceptives inhibit final follicular selection and ovulation? A) They selectively block FSH secretion, preventing granulosa cell activation. B) They suppress LH secretion via negative feedback, preventing the LH surge necessary for ovulation. C) They inhibit theca cell function, stopping androgen production required for estrogen synthesis. D) They desensitize ovarian follicles to gonadotropins, making ovulation impossible. E) They prevent the corpus luteum from forming, thereby stopping progesterone production.
Answer: B ✅ Rationale: Final follicular selection and ovulation require an LH surge, which is suppressed by hormonal contraceptives through negative feedback on GnRH and LH secretion. (A) is incorrect because FSH suppression occurs, but early folliculogenesis is still gonadotropin-independent. (C) is incorrect because theca cells still produce androgens, but estrogen synthesis is limited by low FSH activity. (D) is incorrect because follicles remain responsive, but ovulation is blocked by LH suppression. (E) is incorrect because the corpus luteum only forms post-ovulation, which is already prevented by contraceptives.
33
During the proliferative phase of the uterine cycle, estradiol (E2) secretion primarily functions to: A) Maintain the corpus luteum and prevent ovulation B) Suppress FSH secretion to prevent follicular maturation C) Stimulate endometrial growth and vascularization in preparation for implantation D) Inhibit progesterone secretion to prevent early secretory phase changes E) Prevent development of secretory glands in the endometrium
Answer: C ✅ Rationale: Estradiol (E2) is secreted by the dominant follicle during the late proliferative phase and stimulates endometrial thickening and vascularization to prepare for potential implantation. (A) is incorrect because the corpus luteum forms only after ovulation. (B) is incorrect because FSH is suppressed primarily by inhibins, not estradiol alone. (D) is incorrect because progesterone rises in the secretory phase after ovulation. (E) is incorrect because secretory gland development occurs during the secretory phase, which is progesterone-driven.
34
What is the primary function of progesterone (P) during the secretory phase of the uterine cycle? A) Stimulates continued endometrial hyperplasia, thickening the lining B) Inhibits estradiol-induced hyperplasia and stimulates secretory gland development C) Directly triggers ovulation by increasing LH secretion D) Suppresses endometrial gland function to prevent pregnancy E) Prevents corpus luteum formation and maintains follicular dominance
Answer: B ✅ Rationale: Progesterone (P), secreted from the corpus luteum, plays a protective role by inhibiting estradiol-induced hyperplasia and stimulating the development of secretory glands necessary for implantation. (A) is incorrect because excessive hyperplasia can lead to endometrial pathology, which progesterone prevents. (C) is incorrect because LH, not progesterone, triggers ovulation. (D) is incorrect because secretory gland function is necessary for implantation. (E) is incorrect because progesterone is secreted by the corpus luteum, not involved in its prevention.
35
What is the major risk associated with unopposed estrogen therapy (E2 alone) in postmenopausal hormone replacement therapy (HRT)? A) Increased risk of osteoporosis due to excessive endometrial atrophy B) Higher likelihood of ovarian failure due to excess follicular stimulation C) Endometrial hyperplasia and increased risk of endometrial cancer D) Suppression of FSH and LH, leading to early menopause symptoms E) Increased progesterone production, leading to corpus luteum maintenance
Answer: C ✅ Rationale: Unopposed estradiol (E2) can lead to excessive endometrial hyperplasia, increasing the risk of endometrial cancer. This is why progesterone is combined with estrogen in HRT to counteract hyperplasia and reduce cancer risk. (A) is incorrect because excessive atrophy is more associated with progesterone dominance, not unopposed estrogen. (B) is incorrect because ovarian failure is due to natural aging, not estrogen therapy. (D) is incorrect because FSH and LH suppression occurs naturally in menopause, not due to HRT alone. (E) is incorrect because progesterone is added to HRT, not produced in excess due to estrogen alone.
36
Which of the following best describes Estradiol (E2)? A) Predominant estrogen during reproductive years, secreted by ovarian follicles B) Predominant during pregnancy, produced mainly by the placenta C) Dominant after menopause, synthesized from androgens D) Weak estrogen found only in the adrenal cortex E) Synthetic estrogen used in hormone replacement therapy
Answer: A ✅ Rationale: Estradiol (E2) is the most potent natural estrogen and is predominant during the reproductive years. It is produced mainly by the ovarian follicles. (B) is incorrect because Estriol (E3), not E2, is the primary estrogen during pregnancy. (C) is incorrect because Estrone (E1) is predominant after menopause. (D) is incorrect because E2 is synthesized primarily in the ovaries, not just the adrenal cortex. (E) is incorrect because E2 is naturally occurring, not a synthetic estrogen.
37
Which estrogen is predominant after menopause, and how is it synthesized? A) Estrone (E1), synthesized from androgens via aromatase in adipose tissue B) Estriol (E3), produced by the placenta post-menopause C) Estradiol (E2), continuously produced by the ovaries after menopause D) Synthetic estrogens, primarily used to replace natural estrogen E) Estrone (E1), directly synthesized by ovarian follicles post-menopause
Answer: A ✅ Rationale: Estrone (E1) is the predominant estrogen after menopause. Since ovarian function declines, E1 is synthesized from androgens (androstenedione) via aromatase in adipose tissue. (B) is incorrect because E3 is mainly produced during pregnancy, not after menopause. (C) is incorrect because E2 declines significantly post-menopause. (D) is incorrect because while synthetic estrogens can be used in HRT, they are not a natural dominant estrogen. (E) is incorrect because ovarian follicle function ceases post-menopause.
38
Which statement correctly describes Estriol (E3)? A) Weak estrogen that is the predominant form during pregnancy, produced by the placenta B) The most potent estrogen, maintaining endometrial growth during the reproductive years C) Predominant estrogen in postmenopausal women, synthesized in adipose tissue D) A synthetic estrogen used in oral contraceptives E) A non-functional estrogen, present only in trace amounts in all life stages
Answer: A ✅ Rationale: Estriol (E3) is the weakest natural estrogen and is predominant during pregnancy, produced by the placenta. (B) is incorrect because E2, not E3, is the most potent estrogen and dominates during reproductive years. (C) is incorrect because E1 is the predominant estrogen post-menopause. (D) is incorrect because E3 is not a synthetic estrogen used in contraceptives. (E) is incorrect because E3 plays an essential role in pregnancy.
39
Which of the following best describes estrogen’s function in the menstrual cycle? A) Stimulates endometrial growth and vascularization during the proliferative phase B) Maintains the corpus luteum after ovulation C) Directly triggers ovulation by causing the LH surge D) Suppresses thin cervical mucus production to prevent sperm transport E) Inhibits uterine growth to prevent hyperplasia
Answer: A ✅ Rationale: Estrogen (E2) promotes endometrial thickening, hyperplasia, and vascularization during the proliferative phase. (B) is incorrect because the corpus luteum is maintained by LH and later by hCG, not estrogen. (C) is incorrect because LH, not estrogen alone, directly triggers ovulation. (D) is incorrect because estrogen increases thin cervical mucus to facilitate sperm transport. (E) is incorrect because estrogen stimulates, not inhibits, uterine growth.
40
How does estrogen affect lipid metabolism? A) Increases LDL and decreases HDL B) Increases HDL and decreases LDL C) Has no effect on lipid metabolism D) Promotes cholesterol excretion in bile, reducing gallstone risk E) Inhibits RAAS, lowering blood pressure
Answer: B ✅ Rationale: Estrogen has a cardioprotective effect by increasing HDL ("good" cholesterol) and decreasing LDL ("bad" cholesterol). (A) is incorrect because estrogen does the opposite, improving lipid profiles. (C) is incorrect because estrogen plays a major role in lipid regulation. (D) is incorrect because estrogen actually increases cholesterol saturation in bile, raising gallstone risk. (E) is incorrect because estrogen increases blood pressure via the RAAS system, not lowers it.
41
Why do high estrogen levels (e.g., in oral contraceptives or pregnancy) increase the risk of thrombosis? A) Estrogen inhibits platelet function, leading to clot formation B) Estrogen causes vasoconstriction, reducing blood flow and increasing clot risk C) Estrogen increases hepatic production of clotting factors, promoting thrombosis D) Estrogen suppresses fibrinogen production, leading to a hypercoagulable state E) Estrogen directly activates platelets, leading to spontaneous clot formation
Answer: C ✅ Rationale: Estrogen stimulates the liver to produce more clotting factors (e.g., factors II, VII, IX, and X), increasing the risk of thromboembolism. (A) is incorrect because estrogen does not directly inhibit platelets. (B) is incorrect because estrogen mainly affects coagulation, not vasoconstriction. (D) is incorrect because fibrinogen increases, not decreases, in response to estrogen. (E) is incorrect because estrogen’s effect is primarily on clotting factors, not direct platelet activation.
42
Which of the following correctly describes the role of estrogen (E2) and its physiological effects throughout a woman's lifespan? A) Estrogen exerts negative feedback on gonadotropins, suppressing FSH and LH, but its decline during menopause leads to increased gonadotropin levels and irregular menstrual cycles. B) Estrogen stimulates bone resorption, contributing to osteoporosis risk in menopause. C) Unopposed estrogen therapy is protective against endometrial cancer and is preferred in postmenopausal women. D) Estrogen levels are highest during perimenopause due to follicular atresia increasing E2 secretion. E) Estrogen has no significant effect on thromboembolic risk or breast cancer development.
Answer: A ✅ Rationale: Estrogen (E2) exerts negative feedback on FSH and LH, maintaining menstrual cycle regularity. As estrogen declines in menopause, FSH and LH increase due to reduced negative feedback, leading to irregular cycles and eventual cessation of menstruation. (B) is incorrect because estrogen inhibits, rather than promotes, bone resorption—its decline in menopause increases osteoporosis risk. (C) is incorrect because unopposed estrogen therapy increases the risk of endometrial cancer. (D) is incorrect because estrogen levels decline during perimenopause due to follicular atresia. (E) is incorrect because estrogen increases the risk of thromboembolism and breast cancer, particularly with prolonged exposure.
43
One of these molecules is like the big boss of hormones—it’s where all the others come from. It still has a huge side chain attached. Who is it? A) Estradiol (E2) B) Testosterone C) Cholesterol D) Progesterone E) Estrone (E1)
Answer: C – Cholesterol! ✅ Rationale: Cholesterol is the "grandparent" of all steroid hormones. Every other hormone in the diagram comes from cholesterol after enzymes chop off pieces and modify the structure. Look for the big, clunky side chain—that’s the giveaway.
44
After cholesterol loses part of its side chain, it turns into an important intermediate that still has part of the side chain left. What is this molecule called? A) Estriol (E3) B) Pregnenolone C) Androstenedione D) Estradiol (E2) E) Cortisol
Answer: B – Pregnenolone! ✅ Rationale: Pregnenolone is the first real "baby step" after cholesterol gets trimmed down. It still has some of the side chain left but is getting ready to become progesterone, androgens, and eventually estrogens. If you see a structure with a smaller but still visible side chain, it's Pregnenolone.
45
Estrogens have a unique shape that makes them different from all other steroids. What’s the biggest clue that something is an estrogen? A) It has a "doughnut" (aromatic ring) on one side B) It has a long side chain C) It has no hydroxyl (-OH) groups D) It looks like testosterone E) It has three rings instead of four
Answer: A – Aromatic Ring! ✅ Rationale: Estrogens (E1, E2, E3) are the ONLY ones in this whole pathway with an AROMATIC RING (the "doughnut" shape). That’s what aromatase does—it converts androgens (testosterone) into estrogens by making one of the rings flat and stable (aromatic). If you see a flattened ring in the bottom left corner, it’s an estrogen!
46
How do you know if a molecule is still an androgen (like testosterone) and hasn’t turned into estrogen yet? A) It has a methyl group (C19) that hasn’t been chopped off B) It has an aromatic ring C) It has a super long side chain D) It only has three rings instead of four E) It has no hydroxyl (-OH) groups
Answer: A – The C19 Methyl Group! ✅ Rationale: Androgens still have a tiny methyl group on carbon 19 (C19). When aromatase converts testosterone into estrogen, it CLEAVES OFF C19 and flattens the ring into an aromatic structure. If you still see a small extra carbon at the top, it’s an androgen.
47
Imagine you have a bunch of LEGOs, and one of them is super smooth and flat on top, while the others have little bumps sticking out. Which one is estrogen? A) The one that’s super smooth and flat B) The one with a tiny bump on top C) The one with a long tail hanging off D) The one shaped like a spiky ball E) The one that looks like a hotdog
Answer: A – The One That’s Super Smooth and Flat! ✅ Rationale: Estrogen is the ONLY steroid hormone that’s super flat on one side. Other hormones, like testosterone, still have a tiny bump sticking out (which is actually a carbon, but we’re not saying that). Aromatase acts like a LEGO shaver and smooths estrogen out—that’s how you know it’s different.
48
You’re playing with a puzzle, and you need to fit the piece into the hole. If the piece is too big or too small, it won’t fit. Why does estrogen fit perfectly into the estrogen receptor? A) Because the two important green parts (hydroxyls) are the perfect distance apart B) Because estrogen can change its shape like Play-Doh C) Because it sticks inside the receptor like glue D) Because it has a tail that wraps around the receptor like a monkey E) Because it’s small enough to fit in any receptor
Answer: A – The Two Important Parts Are The Right Distance Apart! ✅ Rationale: Estrogen is like a perfectly cut puzzle piece. The two green parts (hydroxyls, but we won’t call them that) are exactly the right distance apart to fit in the receptor. If they were too close or too far, the receptor wouldn’t recognize estrogen.
49
Imagine you have a book, and you need to close the cover over a piece of paper inside. If the paper is flat, the book closes easily. But if the paper is bumpy, the book won’t shut. Why does estrogen need to be super flat? A) So the receptor can fold over it and turn on important genes B) So it can roll around like a marble inside the cell C) So it can bounce off the walls and move around faster D) So it can hide from other hormones E) So it can stack up inside the cell like pancakes
Answer: A – So The Receptor Can Fold Over It! ✅ Rationale: Estrogen’s flat shape lets the receptor close over it like a book. If estrogen were bumpy, the receptor wouldn’t shut properly and couldn’t turn on the genes it needs to. It’s like trying to close a book with a crumpled piece of paper inside—it just won’t work!
50
Which of the following structural changes must occur for an androgen to be converted into an estrogen? A) A-ring becomes aromatic, and C19 methyl is removed B) Double bond added to ring B C) Hydroxyl (-OH) group added at C21 D) The side chain is extended by two carbons E) Addition of a ketone at C3
Answer: A ✅ Rationale: The A-ring must be aromatic → This is the key difference between androgens and estrogens. C19 methyl is removed → This is why estrogens are missing the tiny bump that testosterone has.
51
Which of the following is required for aromatase to convert testosterone to estradiol? A) 3 O₂ molecules and 3 NADPH B) Addition of a sulfate group C) Activation by FSH D) Reduction of C3 ketone to an alcohol E) Direct hydrolysis of the A-ring
Answer: A ✅ Rationale: Aromatase is a CYP450 enzyme, meaning it requires oxygen and NADPH. The reaction sequence involves oxidation → oxidation → elimination → aromatization.
52
A patient is receiving an aromatase inhibitor for estrogen-sensitive breast cancer. Which of the following would be expected? A) Increased testosterone and androstenedione levels B) Increased estradiol and estrone levels C) Decreased androgen production D) Direct inhibition of estrogen receptors E) Increased levels of cortisol
Answer: A ✅ Rationale: If aromatase is blocked, androgens (testosterone, androstenedione) cannot convert into estrogens (estradiol, estrone). FSH & LH may increase due to lack of negative feedback from estrogen. This is why aromatase inhibitors are used in postmenopausal breast cancer treatment.
53
Question 1: What Do Aromatase Inhibitors Actually Bind To? Aromatase inhibitors work by directly binding to what part of the enzyme? A) The iron (Fe) in the active site of aromatase B) The oxygen of the androgen substrate C) The hydrophobic pocket of the estrogen receptor D) The sulfur bridge in the enzyme E) The phosphate group of NADPH
Answer: A – The Iron in the Active Site! ✅ Rationale: Aromatase inhibitors mimic oxygen’s interaction with the heme (Fe) site in aromatase, preventing oxidation. This stops androgens from converting into estrogens.
54
Which part of an aromatase inhibitor’s structure is critical for blocking the enzyme? A) The triazole (N3) ring B) The chloride (Cl) group C) The cyanide (CN) groups D) The benzene ring E) The hydroxyl (-OH) group
Answer: A – The Triazole (N3) Ring! ✅ Rationale: The triazole (N3) ring binds directly to Fe, stopping the reaction. Other groups (like cyanide or benzene) help selectivity but do NOT block aromatase directly.
55
Aromatase inhibitors share a mechanism with antifungal drugs because they: A) Use nitrogen-containing azole rings to block CYP450 enzymes B) Bind to fungal membranes and disrupt cell walls C) Inhibit fungal RNA transcription D) Reduce fungal steroid synthesis without affecting human hormones E) Block fungal glucose metabolism
Answer: A – Azole Rings Block CYP450 Enzymes! ✅ Rationale: Both antifungals and aromatase inhibitors target CYP450 enzymes using azole rings (N3 groups). Antifungals block ergosterol synthesis, while aromatase inhibitors block estrogen synthesis.
56
Which of the following statements is TRUE about estrone (E1) and estradiol (E2)? A) Estradiol (E2) is the most potent estrogen and directly activates the estrogen receptor. B) Estrone (E1) is the most potent estrogen at the receptor level. C) Estrone (E1) is never converted into estradiol (E2) in the body. D) Estradiol (E2) is the dominant estrogen during menopause. E) Estrone (E1) is only present in circulation and does not store locally.
Answer: A – Estradiol (E2) is the most potent estrogen! ✅ Rationale: Estradiol (E2) is the strongest activator of the estrogen receptor. Estrone (E1) is a weaker form that can be converted into estradiol locally. During menopause, estrone (E1) is dominant, NOT estradiol (E2).
57
Which of the following best explains why estriol (E3) is the most stable for oral administration? A) Estriol (E3) has three hydroxyl (-OH) groups, making it more hydrophilic and resistant to metabolism. B) Estriol (E3) binds most strongly to the estrogen receptor, making it last longer. C) Estriol (E3) cannot be metabolized in the liver. D) Estradiol (E2) has the longest half-life of all estrogens, making it the best oral option. E) Estrone (E1) is the most lipophilic, making it best for oral absorption.
Answer: A – Estriol (E3) is hydrophilic and survives oral metabolism! ✅ Rationale: Estriol (E3) has THREE hydroxyl (-OH) groups, making it more resistant to first-pass metabolism. Estradiol (E2) and estrone (E1) get metabolized faster when taken orally. This is why estriol is often used in hormone therapy despite being the weakest estrogen.
58
Which enzyme is responsible for converting estrone (E1) to estradiol (E2)? A) Estradiol dehydrogenase B) Aromatase C) 5-alpha reductase D) CYP450 3A4 E) Glucuronosyltransferase
Answer: A – Estradiol Dehydrogenase! ✅ Rationale: Estradiol dehydrogenase catalyzes the reversible conversion between E1 (estrone) and E2 (estradiol). Aromatase is involved in androgen → estrogen conversion, NOT between estrogens. 5-alpha reductase is for testosterone → DHT conversion, NOT estrogens.
59
Which structural modification is responsible for ethinyl estradiol’s improved bioavailability? A) Alkyne at C17 prevents oxidation and ketone formation B) Addition of a sulfate group at C3 improves water solubility C) Extra hydroxyl (-OH) group makes it more active at the receptor D) Increased lipophilicity allows better blood-brain barrier penetration E) Aromatic ring is removed, making metabolism slower
Answer: A – Alkyne at C17 Prevents Metabolism! ✅ Rationale: The alkyne stops two things from happening: Prevents oxidation at the aromatic ring. Prevents hydroxyl (-OH) at C17 from turning into a ketone. This means the drug stays active longer = better contraception.
60
Why does tamoxifen block estrogen in breast tissue but not in bone? A) Its side chain slightly changes the receptor shape, preventing full activation in some tissues. B) It has a lower binding affinity for the estrogen receptor in breast tissue. C) Tamoxifen is only active in postmenopausal women. D) Tamoxifen causes downregulation of estrogen receptor production. E) The liver metabolizes tamoxifen into an inactive form before it reaches bone.
Answer: A – Side Chain Changes the Receptor Shape! ✅ Rationale: SERMs like tamoxifen don’t just “block” estrogen; they slightly change how the receptor behaves. This prevents the receptor from fully activating in some tissues (like breast), but still allows partial activation in others (like bone).
61
Raloxifene is used to prevent osteoporosis while tamoxifen is not. Why? A) Raloxifene blocks estrogen in breast & uterus but activates it in bone. B) Raloxifene is a stronger estrogen agonist than tamoxifen. C) Raloxifene is rapidly metabolized, so it only works in bone. D) Tamoxifen has no effect on the estrogen receptor in bone tissue. E) Raloxifene increases calcium absorption more than tamoxifen.
Answer: A – Raloxifene Blocks Breast/Uterus but Activates Bone! ✅ Rationale: Raloxifene = SERM that acts as an antagonist in breast/uterus but a partial agonist in bone. This makes it great for osteoporosis prevention without increasing breast cancer risk.
62
Which of the following structural features is required for progesterone activity at its receptor? A) Flat A-ring with all trans connections B) Aromatic A-ring C) Hydroxyl (-OH) at C3 instead of a ketone D) Extended side chain at C17 E) U-shaped conformation
Answer: A – Flat A-ring with all trans connections! ✅ Rationale: Progesterone must be FLAT to bind the nuclear receptor! If it bends (U-shape), it’s inactive! Aromatization (B) would turn it into estrogen. Hydroxyl (C) instead of a ketone would kill activity.
63
Which of the following explains why synthetic progestins have longer half-lives than natural progesterone? A) They resist metabolism at C17, preventing rapid inactivation. B) They bind stronger to the estrogen receptor. C) They lack a double bond in the A-ring. D) They are converted into estrone before excretion. E) They avoid metabolism by being stored in adipose tissue.
Answer: A – They Resist Metabolism at C17! ✅ Rationale: C17 is the main site of metabolism in progesterone. Synthetic progestins modify C17 to prevent breakdown → longer duration.
64
Why do some synthetic progestins (like certain older generations) cause androgenic side effects? A) They have structural similarity to testosterone, allowing them to bind androgen receptors. B) They directly increase testosterone production. C) They convert into testosterone in the liver. D) They act as estrogen receptor agonists in some tissues. E) They cause LH surges, leading to increased androgen synthesis.
Answer: A – Structural Similarity to Testosterone! ✅ Rationale: Some progestins have a structure close to testosterone → they accidentally bind androgen receptors. This leads to side effects like acne, hirsutism, and masculinization.
65
Which structural feature of Medroxyprogesterone Acetate increases oral bioavailability and duration of action? A) C17 acetate (prodrug) + C6 methyl (prevents metabolism) B) Spiro ring that prevents oxidation C) Hydroxyl (-OH) at C6 increases receptor binding D) Aromatic A-ring improves solubility E) Fluorine group increases membrane permeability
Answer: A – C17 Acetate (Prodrug) + C6 Methyl (Prevents Metabolism)! ✅ Rationale: C17 Acetate = Makes it a prodrug → Activated in the liver. C6 Methyl = Blocks metabolism → Prevents U-shape bending → Longer half-life.
66
Drospirenone is unique among progestins because it reduces bloating and water retention. Why? A) It blocks mineralocorticoid receptors (anti-MR activity). B) It increases sodium excretion via aldosterone activation. C) It has a longer half-life than progesterone. D) It increases estradiol conversion, reducing estrogenic effects. E) It binds to androgen receptors, decreasing testosterone levels.
Answer: A – It Blocks Mineralocorticoid Receptors (Anti-MR Activity)! ✅ Rationale: Drospirenone is structurally similar to eplerenone (a spironolactone analog). It blocks aldosterone’s effects → Less sodium/water retention → Less bloating.
67
Drospirenone shares anti-mineralocorticoid properties with spironolactone due to which key structural feature? A) Spiro Ring B) C6 Methyl Group C) Aromatic A-Ring D) Fluorine Substitution E) C17 Acetate Prodrug Form
A ✅ Rationale: Spiro Ring = What makes drospirenone act like eplerenone. It prevents mineralocorticoid receptor activation (blocks aldosterone effects).
68
Question 1: How Do GnRH Antagonists Work? How do Elagolix and Relugolix function to reduce estrogen levels? A) They competitively bind the GnRH receptor, blocking natural GnRH from stimulating FSH & LH release. B) They desensitize the GnRH receptor, causing receptor downregulation. C) They increase LH and FSH release, indirectly reducing estrogen production. D) They block estrogen receptors in the pituitary gland. E) They increase testosterone and estradiol levels through negative feedback.
Answer: A – Competitive GnRH Receptor Antagonists! ✅ Rationale: These drugs immediately block GnRH receptors, preventing FSH & LH release.
69
Which structural feature of Elagolix & Relugolix is essential for binding the GnRH receptor? A) Aromatic rings and heterocyclic core B) Sulfate group for receptor activation C) Extended peptide backbone for slow receptor activation D) Open-chain conformation for enzyme interaction E) Phosphate groups for improved water solubility
Answer: A – Aromatic Rings & Heterocyclic Core! ✅ Rationale: Both Elagolix & Relugolix contain multiple aromatic rings, which help bind to the receptor. The heterocyclic core (purple region in the image) is the main functional part that blocks GnRH binding.
70
Elagolix and Relugolix are orally bioavailable, unlike peptide-based GnRH drugs (e.g., leuprolide). Why? A) They are small molecules, not peptides, allowing them to survive GI metabolism. B) They are administered with proton pump inhibitors to prevent stomach acid breakdown. C) They require enzymatic activation in the liver before becoming active. D) They are rapidly degraded but have high first-pass activation. E) They bind albumin in plasma, increasing half-life.
Answer: A – Small Molecules, Not Peptides! ✅ Rationale: Leuprolide (and other GnRH agonists) are peptides, so they get destroyed in the stomach. Elagolix & Relugolix are non-peptide small molecules → stable for oral use.
71
Question 1: Which Contraceptive Method is Least Affected by Adherence? Which contraceptive method is least dependent on patient adherence for effectiveness? A) Implant or IUD B) Oral contraceptive pills C) Patch or ring D) Condoms E) Fertility awareness
Answer: A – Implant or IUD! ✅ Rationale: IUDs and implants require no daily adherence, so they have the lowest real-world failure rate. Pills, patches, and rings depend on proper use → Higher failure rates in typical use.
72
What is the most common reason for contraceptive failure in real-world use? A) Inconsistent or incorrect use of the method (adherence issues) B) Inherent failure rate of the method C) Lack of partner cooperation D) Cost and access to contraception E) Pharmacokinetic variability of hormone levels
Answer: A – Inconsistent or Incorrect Use! ✅ Rationale: Typical use failure rates are much higher than perfect use rates due to user error. This is especially true for methods requiring daily or regular action (pills, patches, rings).
73
Which of the following methods relies most heavily on perfect use for effectiveness? A) Progestin-only pills B) Copper IUD C) Implant D) Depo-Provera shot E) Vaginal ring
Answer: A – Progestin-Only Pills! ✅ Rationale: Progestin-only pills MUST be taken at the exact same time every day. A few hours late = Risk of ovulation. Other options (IUD, implant, shot) don’t require such strict adherence.
74
Question 1: What is the Primary Pregnancy Prevention Mechanism? Which of the following is the most important mechanism by which progestin prevents pregnancy? A) Suppresses ovulation by blocking the LH surge B) Thickens cervical mucus C) Slows fallopian tube motility D) Thins the endometrial lining E) Inhibits FSH release
Answer: A – Blocks the LH Surge! ✅ Rationale: No LH surge = No ovulation. This is the most critical effect of progestin in preventing pregnancy. Other effects (like mucus thickening) add backup but aren’t the primary method.
75
Why is estrogen combined with progestin in contraceptives? A) To stabilize the endometrium and prevent breakthrough bleeding B) To increase cervical mucus viscosity C) To increase fallopian tube motility D) To increase FSH and LH secretion E) To enhance ovulation suppression
Answer: A – Endometrial Stabilization! ✅ Rationale: Estrogen’s main job is cycle regulation, NOT blocking ovulation. Prevents breakthrough bleeding and spotting. Progestin alone can cause cycle instability, which is why estrogen is added.
76
Progestin-only contraceptive pills have a higher failure rate than combined contraceptives because: A) They do not reliably suppress ovulation. B) They do not thicken cervical mucus. C) They cause increased tubal motility. D) They increase endometrial thickness. E) They increase FSH secretion.
Answer: A – They Do Not Reliably Suppress Ovulation! ✅ Rationale: Progestin-only pills rely more on mucus thickening & tubal effects, but ovulation suppression isn’t as strong. This makes them less effective than combined contraceptives.
77
Question 1: What is the Most Recommended Oral Contraceptive Type? Which oral contraceptive pill regimen is most commonly recommended? A) Monophasic (one dose of P/E used throughout the cycle) B) Biphasic (two doses of P/E throughout the cycle) C) Triphasic (three doses of P/E throughout the cycle) D) Progestin-only pill E) Extended regimen
Answer: A – Monophasic! ✅ Rationale: One steady dose = most stable cycle & most commonly used. Multiphasic regimens exist but aren’t the go-to recommendation.
78
A patient wants to completely eliminate their period with birth control. Which regimen is the best option? A) Extended regimen (continuous use, no inactive pills) B) Monophasic pills C) Progestin-only pills D) Shortened placebo (24/4) E) Triphasic pills
Answer: A – Extended Regimen! ✅ Rationale: Skipping hormone-free intervals (no inactive pills) = No period at all. Most effective for cycle suppression.
79
Which of the following is NOT a proven side effect of combined oral contraceptives? A) Breast tenderness B) Nausea C) Headache D) Weight gain E) All of the above are true side effects
Answer: D – Weight Gain! ✅ Rationale: Weight gain has NOT been consistently proven in clinical studies.
80
Which of the following is true about progestin-only oral contraceptives? A) They require strict adherence to timing (same time every day) to be effective. B) They are more effective than combined oral contraceptives. C) They contain both estrogen and progestin. D) They are the most commonly recommended contraceptive type. E) They are preferred for people who want to regulate their menstrual cycle.
Answer: A – Require Strict Timing! ✅ Rationale: Progestin-only pills MUST be taken at the same time every day. Even a few hours late = higher failure risk.
81
A patient on combined oral contraceptives reports breakthrough bleeding during the first half of the cycle. What is the most likely cause? A) Estrogen dose is too low B) Progestin dose is too low C) Estrogen dose is too high D) Patient is taking the pill incorrectly E) Progestin is too androgenic
Answer: A – Estrogen dose is too low! ✅ Rationale: Breakthrough bleeding in the follicular phase (first half of the cycle) = Not enough estrogen to stabilize endometrium.
82
Which of the following patients is at the highest risk of MI/stroke if started on combined oral contraceptives? A) 39-year-old woman who smokes a pack per day B) 24-year-old woman who smokes occasionally C) 28-year-old woman with no medical history D) 32-year-old woman with a history of ovarian cysts E) 35-year-old woman with normal blood pressure and no smoking history
Answer: A – 39-year-old woman who smokes! ✅ Rationale: Smoking + Age >35 = Absolute Contraindication (High Stroke/MI Risk).
83
A 27-year-old female on combined oral contraceptives reports severe unilateral leg pain and swelling. What is the most appropriate next step? A) Discontinue oral contraceptive immediately and evaluate for deep vein thrombosis (DVT). B) Switch to a progestin-only pill to reduce clot risk. C) Reassure the patient that muscle cramps are common. D) Prescribe ibuprofen and encourage hydration. E) Increase estrogen dose to improve cycle control.
Answer: A – Stop OCP & Evaluate for DVT! ✅ Rationale: Severe unilateral leg pain = Possible DVT. Oral contraceptives must be stopped immediately, and patient needs urgent evaluation.
84
A patient taking spironolactone for acne is prescribed a combined hormonal contraceptive. Which progestin should be avoided? A) Drospirenone B) Levonorgestrel C) Norethindrone D) Desogestrel E) Medroxyprogesterone
Answer: A – Drospirenone! ✅ Rationale: Drospirenone has anti-mineralocorticoid effects, like spironolactone → can increase potassium too much!
85
Which of the following symptoms suggests progestin deficiency in a patient on combined hormonal contraception? A) Breakthrough bleeding in the luteal phase B) Breakthrough bleeding in the follicular phase C) Nausea and vomiting D) Breast tenderness E) Fatigue and decreased libido
Answer: A – Breakthrough Bleeding in Luteal Phase! ✅ Rationale: Too little progestin = Endometrial instability → Luteal phase spotting. 📌 Exam Trap: Follicular phase breakthrough bleeding = Estrogen deficiency, NOT progestin!
86
Which of the following cancer risks is most significantly reduced by combined hormonal contraception? A) Ovarian and Endometrial Cancer B) Breast Cancer C) Cervical Cancer D) Colon Cancer E) Lung Cancer
Answer: A – Ovarian & Endometrial Cancer! ✅ Rationale: Ovarian cancer risk decreases because CHCs suppress ovulation. Endometrial cancer risk decreases because progestin prevents hyperplasia. 📌 Exam Trap: If they ask about breast cancer risk → CHC does NOT reduce it!
87
Question 1: How Many Missed Pills Require Backup Contraception? A patient on a combined oral contraceptive missed two pills (48+ hours) in the middle of the pack. What should she do? A) Take the most recent missed pill ASAP, discard others, and use backup for 7 days. B) Take both missed pills ASAP and continue as normal with no backup needed. C) Restart the pack from the beginning and use backup for 14 days. D) Skip the missed pills and resume with today’s pill. E) Use emergency contraception immediately, regardless of cycle timing.
Answer: A – Take the most recent missed pill ASAP & Use Backup for 7 Days! ✅ Rationale: 2+ missed pills = 48+ hours since last dose = Backup needed. 📌 Exam Trap: Don’t take ALL missed pills – just take the most recent one and discard the rest.
88
Which of the following antibiotics is known to reduce the effectiveness of combined oral contraceptives? A) Rifampin B) Amoxicillin C) Azithromycin D) Ciprofloxacin E) Doxycycline
Answer: A – Rifampin! ✅ Rationale: Rifampin induces hepatic enzymes, accelerating estrogen/progestin metabolism. 📌 Exam Trap: No other antibiotics have strong evidence of reducing OCP effectiveness!
89
A patient on combined oral contraceptives reports breakthrough bleeding during the first two weeks of her cycle (follicular phase). What is the most appropriate adjustment? A) Increase estrogen dose (up to 35 mcg). B) Change to a progestin with higher androgen activity. C) Increase progestin dose. D) Stop contraception and use an IUD instead. E) Add a week of placebo pills to increase cycle regularity.
Answer: A – Increase Estrogen Dose! ✅ Rationale: Follicular phase breakthrough bleeding = Not enough estrogen stabilizing the endometrium. 📌 Exam Trap: Luteal phase bleeding (after day 14) = Change Progestin, NOT Estrogen!
90
A patient using NuvaRing accidentally removes it and forgets to put it back in for 5 hours. What should she do? A) Rinse & Reinsert ASAP, No Backup Needed. B) Rinse & Reinsert, Use Backup for 7 Days. C) Start a New Ring Immediately & Use Backup for 7 Days. D) Wait for Her Next Period to Restart a New Ring. E) Take an Emergency Contraceptive Pill.
Answer: B – Rinse & Reinsert, Use Backup for 7 Days! ✅ Rationale: Ring out for >3 hours = Backup Needed for 7 Days. 📌 Exam Trap: If the ring was out for <3 hours, backup contraception wouldn’t be needed!
91
Which of the following contraceptive methods is associated with the highest risk of venous thromboembolism (VTE)? A) Transdermal Patch (Xulane, Twirla) B) Vaginal Ring (NuvaRing) C) Combined Oral Contraceptive (Pill) D) Progestin-Only Pill (POP) E) Depo Medroxyprogesterone (Depo Shot)
Answer: A – Transdermal Patch (Xulane, Twirla)! ✅ Rationale: The Patch has higher estrogen exposure than oral/vaginal forms = Increased VTE Risk. 📌 Exam Trap: Don’t pick the pill! Patch > Pill for VTE risk.
92
A patient is considering different contraceptive options but wants to get pregnant soon after stopping contraception. Which method should she avoid? A) Depo Medroxyprogesterone (Depo-Provera) B) Combined Oral Contraceptive C) Vaginal Ring D) Copper IUD E) Progestin-Only Pill
Answer: A – Depo-Provera! ✅ Rationale: Delayed ovulation return (up to 10 months after stopping). 📌 Exam Trap: Oral contraceptives, rings, and IUDs allow rapid fertility return!
93
Question 1: Which IUD Is Best for a Patient With Heavy Bleeding? A 35-year-old female has heavy menstrual bleeding and is interested in long-term contraception. Which of the following options is most appropriate?
Answer: A – Mirena IUD (Levonorgestrel)! ✅ Rationale: Hormonal IUD (Mirena) leads to amenorrhea, which is ideal for heavy bleeding. Copper IUD makes bleeding WORSE.
94
Which of the following contraceptive methods is most effective for emergency contraception? A) Copper IUD B) Levonorgestrel IUD (Mirena) C) Plan B (Levonorgestrel Pill) D) Nexplanon E) Phexxi
Answer: A – Copper IUD! ✅ Rationale: Copper IUD can be inserted up to 5 days after unprotected sex. 📌 Exam Trap: Plan B is less effective than a Copper IUD for emergency contraception.
95
A 27-year-old female is considering Nexplanon for contraception. Which of the following is the most common side effect? A) Irregular bleeding that does not improve over time. B) Weight gain. C) Delayed return to fertility. D) Nausea and vomiting. E) Increased risk of blood clots.
Answer: A – Irregular Bleeding That Does NOT Improve Over Time! ✅ Rationale: Nexplanon causes persistent irregular bleeding (unlike the IUD, which stabilizes over time). 📌 Exam Trap: Nexplanon does NOT delay fertility return!
96
Which of the following patients should NOT be prescribed combined hormonal contraceptives (CHCs)? A) A 29-year-old with well-controlled hypertension (BP 130/85). B) A 42-year-old smoker who smokes 10 cigarettes per day. C) A 35-year-old smoker who smokes 20 cigarettes per day. D) A 25-year-old with a history of migraines without aura. E) A 19-year-old postpartum 4 weeks ago.
Answer: C – A 35-year-old smoker who smokes 20 cigarettes per day! ✅ Rationale: CHCs are contraindicated in patients 35+ years old who smoke ≥15 cigarettes/day due to VTE risk! Patients <35 years old who smoke are NOT automatically contraindicated. 📌 Exam Trap: They will try to trick you with a 42-year-old smoker who smokes <15/day (NOT a contraindication!).
97
A 31-year-old female (BMI 35) had unprotected sex 3 days ago. She wants the most effective emergency contraception. What is the best option? A) Levonorgestrel (Plan B One-Step). B) Ulipristal Acetate (Ella). C) Copper IUD. D) Progestin-Only Pill. E) Combined Oral Contraceptive (Yuzpe Method).
Answer: B – Ulipristal Acetate (Ella)! ✅ Rationale: More effective for overweight/obese patients compared to Plan B. Plan B loses efficacy at higher BMI. 📌 Exam Trap: They will try to trick you by offering Plan B as an option.
98
A pregnant woman was exposed to a known teratogen at Week 5 of pregnancy. What is the fetus at the highest risk for? A) Neural Tube Defects. B) Preterm Birth. C) Major Structural Malformations. D) Cognitive Deficits Without Malformations. E) Fetal Growth Restriction Without Major Defects.
Answer: C – Major Structural Malformations! ✅ Rationale: Weeks 3-8 (Organogenesis) is the most sensitive period for teratogen-induced malformations. 📌 Exam Trap: Neural Tube Defects occur very early (Week 3-4), but Organogenesis (Week 3-8) is when major birth defects occur!
99
Question 1: Which Fetal Development Stage Is Most Vulnerable to Teratogens? A pregnant woman was exposed to a known teratogen at Week 5 of pregnancy. What is the fetus at the highest risk for? A) Neural Tube Defects. B) Preterm Birth. C) Major Structural Malformations. D) Cognitive Deficits Without Malformations. E) Fetal Growth Restriction Without Major Defects.
Answer: C – Major Structural Malformations! ✅ Rationale: Weeks 3-8 (Organogenesis) is the most sensitive period for teratogen-induced malformations. 📌 Exam Trap: Neural Tube Defects occur very early (Week 3-4), but Organogenesis (Week 3-8) is when major birth defects occur!
100
Which of the following teratogens is most likely to cause significant fetal defects throughout the ENTIRE length of pregnancy? A) Isotretinoin (Accutane). B) Thalidomide. C) Alcohol. D) Methotrexate. E) Tetracycline.
Answer: C – Alcohol (Fetal Alcohol Syndrome). ✅ Rationale: Alcohol is teratogenic throughout pregnancy (causes craniofacial dysmorphology, CNS defects, growth retardation). 📌 Exam Trap: They will try to trick you with Isotretinoin, but it primarily affects Weeks 3-8!
101
A drug is known to degrade the SALL4 transcription factor via ubiquitination, leading to limb malformations in exposed fetuses. Which drug is most likely responsible? A) Alcohol. B) Isotretinoin. C) Methotrexate. D) Thalidomide. E) Phenytoin.
Answer: D – Thalidomide! ✅ Rationale: Thalidomide degrades SALL4 → Causes limb reduction defects! 📌 Exam Trap: They will try to trick you by mentioning "gene degradation," but the answer is always Thalidomide for limb defects!
102
Question 1: Which Drug is Safest for a Pregnant Woman With a Mechanical Heart Valve? A 30-year-old woman with a mechanical heart valve is 9 weeks pregnant and requires anticoagulation. Which medication is the best option? A) Warfarin. B) Rivaroxaban. C) LMWH (e.g., Enoxaparin). D) Aspirin Alone.
Answer: C – LMWH! ✅ Rationale: Warfarin is contraindicated due to fetal warfarin syndrome. DOACs (rivaroxaban) are NOT recommended in pregnancy. Aspirin alone is not sufficient. 📌 Exam Trap: They might try to trick you by saying “patient has a mechanical valve,” but warfarin is still teratogenic!
103
A woman took a known teratogen at Week 6 of pregnancy. What is the fetus at the highest risk for? A) Neural Tube Defects. B) Preterm Birth. C) Major Structural Malformations. D) Cognitive Deficits Without Malformations.
Answer: C – Major Structural Malformations! ✅ Rationale: Weeks 3-8 (Organogenesis) is the most critical period for major congenital defects. 📌 Exam Trap: Neural tube defects occur early (Week 3-4), but the BIGGEST malformations happen during Weeks 3-8!
104
A 28-year-old woman was taking a medication for hypertension before realizing she was pregnant. Ultrasound reveals oligohydramnios and lung hypoplasia in the fetus. What medication is most likely responsible? A) Labetalol. B) Hydralazine. C) ACE Inhibitor. D) Methyldopa.
Answer: C – ACE Inhibitor! ✅ Rationale: ACE inhibitors (and ARBs) disrupt fetal kidney development → oligohydramnios, lung hypoplasia, and renal failure. Labetalol and methyldopa are safer alternatives for hypertension in pregnancy. 📌 Exam Trap: They might try to confuse you by listing methyldopa or labetalol as answer choices, but ACE inhibitors are the WORST for fetal kidneys!
105
A pregnant woman was exposed to a known teratogen at Week 6 of pregnancy. What is the fetus at the highest risk for? A) Neural Tube Defects. B) Preterm Birth. C) Major Structural Malformations. D) Cognitive Deficits Without Malformations.
Answer: C – Major Structural Malformations! ✅ Rationale: Weeks 3-8 (Organogenesis) = Highest risk for birth defects!
106
Which of the following teratogens is most likely to cause significant fetal defects throughout the ENTIRE pregnancy? A) Isotretinoin (Accutane). B) Thalidomide. C) Alcohol. D) Methotrexate.
Answer: C – Alcohol (Fetal Alcohol Syndrome). ✅ Rationale: Alcohol is teratogenic the entire pregnancy!
107
A 27-year-old woman with PCOS presents with irregular menses and worsening acne. She does not desire pregnancy at this time. What is the most appropriate first-line treatment? A) Metformin B) Clomiphene Citrate C) Spironolactone D) Combination Oral Contraceptive (CHC) E) Letrozole
Answer: D – CHC (Combination Hormonal Contraceptive). ✅ Rationale: CHCs regulate cycles, reduce hirsutism/acne, and prevent endometrial hyperplasia. Metformin is second-line unless insulin resistance is the main concern. 📌 Exam Trap: Spironolactone must be used with contraception due to teratogenicity!
108
A 30-year-old woman with PCOS is struggling with infertility. She does not want to start medications immediately. What is the most effective non-pharmacologic intervention to restore ovulation? A) Low-carb diet B) Metformin C) 5-10% weight loss D) Clomiphene Citrate E) Progesterone Challenge Test
Answer: C – 5-10% Weight Loss! ✅ Rationale: Weight loss reduces insulin resistance, lowers androgens, and can restore ovulation naturally! 📌 Exam Trap: Metformin is useful but doesn’t directly address the root issue if obesity is a factor!
109
Question 1: Missed Birth Control Pills – What Do You Do? A 22-year-old woman has been taking a combined oral contraceptive (COC). She missed two consecutive pills in Week 2 of her cycle and had unprotected sex yesterday. What is the BEST course of action? A) Take the most recent missed pill ASAP, discard the other missed pill, and use backup contraception for 7 days. B) Take both missed pills immediately and continue the pack as normal, no backup needed. C) Restart a new pack immediately and use emergency contraception. D) Stop the pill for 7 days, then restart a new pack. E) Use emergency contraception and wait until next cycle to restart.
Answer: A – Take the most recent missed pill ASAP, discard the other, and use backup for 7 days. ✅ Rationale: If ≥ 48 hours late, take the most recent missed pill ASAP & continue as normal. Use backup for 7 days if ≥ 48 hours late. Emergency contraception is only needed if missed pills were in Week 1 AND unprotected sex occurred. 📌 Exam Trap: They will try to trick you into thinking every missed pill needs EC – only if Week 1!
110
A 35-year-old woman who has been taking an ethinyl estradiol/norethindrone contraceptive presents with severe unilateral leg pain and swelling. She has a history of smoking and migraines with aura. What is the NEXT best step? A) Prescribe aspirin and continue the pill. B) Switch to a progestin-only pill (POP). C) Switch to a low-dose estrogen contraceptive. D) Advise immediate discontinuation of the COC and evaluate for VTE. E) Prescribe NSAIDs and continue the pill.
Answer: D – STOP the pill immediately and evaluate for VTE. ✅ Rationale: Severe unilateral leg pain = DVT → STOP Estrogen IMMEDIATELY! ACHES Mnemonic: Abdominal pain, Chest pain, Headache, Eye problems, Severe leg pain = RED FLAGS! Smokers > 35 and those with migraines with aura = ABSOLUTE C/I to Estrogen! 📌 Exam Trap: They will try to trick you into “switching to a lower estrogen dose” – NO! STOP immediately!
111
A 29-year-old woman with a BMI of 35 had unprotected sex 3 days ago. She wants emergency contraception but is worried about effectiveness. What is the BEST option for her? A) Levonorgestrel (Plan B). B) Yuzpe method (high-dose COC). C) Copper IUD. D) Ulipristal (Ella). E) Depot medroxyprogesterone acetate (DMPA).
Answer: C – Copper IUD! ✅ Rationale: Most effective emergency contraception, BMI doesn’t matter. Plan B loses effectiveness at BMI > 25. Ulipristal (Ella) loses effectiveness at BMI > 30. 📌 Exam Trap: They will try to trick you into picking Plan B – it’s not effective for high BMI!
112
A 27-year-old woman recently gave birth and wants to start contraception. She is breastfeeding exclusively. Which option is MOST appropriate? A) Combination oral contraceptive (ethinyl estradiol/norethindrone). B) Depot medroxyprogesterone acetate (DMPA). C) Copper IUD. D) Drospirenone-containing combined contraceptive. E) NuvaRing.
Answer: C – Copper IUD. ✅ Rationale: Best option: Non-hormonal (Copper IUD). Progestin-only (DMPA, mini-pill) are safe but may affect milk supply. AVOID Estrogen (combined oral contraceptives, NuvaRing) in the first 6 weeks postpartum! 📌 Exam Trap: They will try to trick you with “low-dose estrogen” – NO estrogen in breastfeeding moms early postpartum!
113
A 32-year-old woman with PCOS has been trying to conceive for over a year without success. She has irregular periods, obesity, and signs of hyperandrogenism. Which of the following is the MOST appropriate first-line treatment for ovulation induction? A) Metformin. B) Clomiphene citrate. C) Letrozole. D) Combined oral contraceptive pill. E) Spironolactone.
Answer: C – Letrozole (Aromatase Inhibitor). ✅ Rationale: 1st line for ovulation induction in PCOS (better than Clomiphene, less risk of multiple births). Metformin helps insulin resistance but is NOT 1st line for ovulation. OCPs and Spironolactone are for hyperandrogenism, NOT for fertility. 📌 Exam Trap: They will try to trick you into picking Clomiphene – Letrozole is preferred!
114
Which of the following patients is the BEST candidate for unopposed estrogen therapy (without progestin)? A) A 54-year-old with an intact uterus and no history of cancer. B) A 60-year-old with a history of breast cancer. C) A 62-year-old who had a hysterectomy 5 years ago. D) A 49-year-old smoker with migraines. E) A 58-year-old with a history of DVT/PE.
Answer: C – A 62-year-old who had a hysterectomy. ✅ Rationale: Unopposed estrogen is only safe in women WITHOUT a uterus! 📌 Exam Trap: They will try to trick you by giving a patient with a uterus (who must take progestin too).
115